Difference between revisions of "2021 Fall AMC 12A Problems/Problem 1"

(Solution 3)
Line 18: Line 18:
 
== Solution 3 ==
 
== Solution 3 ==
 
We have
 
We have
\begin{align*}
+
<cmath>
 
\frac{\left( 2112 - 2021 \right)^2}{169}
 
\frac{\left( 2112 - 2021 \right)^2}{169}
 
& = \frac{91^2}{13^2} \\
 
& = \frac{91^2}{13^2} \\
 
& = 7^2 \\
 
& = 7^2 \\
 
& = 49 .
 
& = 49 .
\end{align*}
+
</cmath>
  
 
Therefore, the answer is \boxed{\textbf{(C) 49}}.
 
Therefore, the answer is \boxed{\textbf{(C) 49}}.

Revision as of 20:46, 25 November 2021

The following problem is from both the 2021 Fall AMC 10A #1 and 2021 Fall AMC 12A #1, so both problems redirect to this page.

Problem

What is the value of $\frac{(2112-2021)^2}{169}$?

$\textbf{(A) } 7 \qquad\textbf{(B) } 21 \qquad\textbf{(C) } 49 \qquad\textbf{(D) } 64 \qquad\textbf{(E) } 91$

Solution 1 (Laws of Exponents)

We have \[\frac{(2112-2021)^2}{169}=\frac{91^2}{169}=\frac{91^2}{13^2}=\left(\frac{91}{13}\right)^2=7^2=\boxed{\textbf{(C) } 49}.\] ~MRENTHUSIASM

Solution 2 (Difference of Squares)

We have \[\frac{(2112-2021)^2}{169}=\frac{91^2}{169}=\frac{(10^2-3^2)^2}{169}=\frac{(10+3)^2(10-3)^2}{169}=\frac{13^2 \cdot 7^2}{13^2}=7^2=\boxed{\textbf{(C) } 49}.\]

Solution 3

We have

\[\frac{\left( 2112 - 2021 \right)^2}{169}
& = \frac{91^2}{13^2} \\
& = 7^2 \\
& = 49 .\] (Error compiling LaTeX. Unknown error_msg)

Therefore, the answer is \boxed{\textbf{(C) 49}}.

~Steven Chen (www.professorchenedu.com)

See Also

2021 Fall AMC 12A (ProblemsAnswer KeyResources)
Preceded by
First Problem
Followed by
Problem 2
1 2 3 4 5 6 7 8 9 10 11 12 13 14 15 16 17 18 19 20 21 22 23 24 25
All AMC 12 Problems and Solutions
2021 Fall AMC 10A (ProblemsAnswer KeyResources)
Preceded by
First Problem
Followed by
Problem 2
1 2 3 4 5 6 7 8 9 10 11 12 13 14 15 16 17 18 19 20 21 22 23 24 25
All AMC 10 Problems and Solutions

The problems on this page are copyrighted by the Mathematical Association of America's American Mathematics Competitions. AMC logo.png